Mathcenter Forum  

Go Back   Mathcenter Forum > คณิตศาสตร์โอลิมปิก และอุดมศึกษา > พีชคณิต
สมัครสมาชิก คู่มือการใช้ รายชื่อสมาชิก ปฏิทิน ข้อความวันนี้

ตั้งหัวข้อใหม่ Reply
 
เครื่องมือของหัวข้อ ค้นหาในหัวข้อนี้
  #1  
Old 17 พฤศจิกายน 2016, 03:50
i^i i^i ไม่อยู่ในระบบ
กระบี่ไว
 
วันที่สมัครสมาชิก: 02 มีนาคม 2014
ข้อความ: 230
i^i is on a distinguished road
Default ขอเเนวทางการพิสูจน์หน่อยครับ

ขอเเนวทางการพิสูจน์เเบบอุปนัยเชิงคณิตศาสตร์หน่อยครับ
ปล. ผมสมมติ ให้ m=6 จะได้ n_1=4 , m=8 จะได้n_1=7
ตรงขั้นตอนเเสดงว่า P(n_1,6) เป็นจริง
เเล้วมาติดตรง P(n_1,k)->P(n_1,k+1) เป็นจริง
ยังไปต่อไม่ถูกครับ ขอคำเเนะนำด้วยครับ
ขอบคุณครับ
รูปภาพที่แนบมาด้วย
 
ตอบพร้อมอ้างอิงข้อความนี้
  #2  
Old 24 พฤศจิกายน 2016, 12:31
i^i i^i ไม่อยู่ในระบบ
กระบี่ไว
 
วันที่สมัครสมาชิก: 02 มีนาคม 2014
ข้อความ: 230
i^i is on a distinguished road
Default

รบกวนขอ Hint หน่อยครับ ผมติดมาหลายวันเเล้วครับ .:T_T:.
ตอบพร้อมอ้างอิงข้อความนี้
  #3  
Old 26 พฤศจิกายน 2016, 18:36
Aquila Aquila ไม่อยู่ในระบบ
บัณฑิตฟ้า
 
วันที่สมัครสมาชิก: 29 ตุลาคม 2013
ข้อความ: 412
Aquila is on a distinguished road
Default

อ้างอิง:
ข้อความเดิมเขียนโดยคุณ i^i View Post
รบกวนขอ Hint หน่อยครับ ผมติดมาหลายวันเเล้วครับ .:T_T:.
โจทย์มันอยากได้ความสัมพันธ์ของ "ค่าเลขชี้กำลังสูงสุด" (ที่เจอใน $m!$)

กับ "จำนวน $m$" จริงป่าวครับ เพราะฉะนั้นพยายามหาเครื่องมือมา "เปรียบเทียบ"

ค่าของเลขชี้กำลังสูงสุดกับค่าของ value $m$

พิสูจน์ให้ได้ก่อนว่าค่ามากที่สุดของเลขชี้กำลัง มันจะเจอใน $p_{1}=2$ ครับ

แล้วใช้เครื่องมือบางอย่าง "โยง" ข้อมูลของ $n_{1}$ (ในตำแหน่ง $p=2$)

เข้าไปเปรียบเทียบกับค่าของ $m$ ครับ

หรือง่ายๆคือ สร้างความสัมพันธ์ของเลขชี้กำลัง $n_{1}-1$ กับ $m$ ให้เปรียบเทียบค่ากันให้ได้

ผมทดจนสุดๆดูแล้วมันไม่ได้ใช้อุปนัยเชิงคณิตศาสตร์ได้ตรงๆครับ ต้องทอนโจทย์ลงมาก่อน

ถ้าไม่ได้จริงๆ แล้ววิธีผมไม่มีปัญหาเดี๋ยวมาพิมพ์ไว้ให้ทีหลังครับ สู้ๆนะครับ

ปล. ปัญหาข้อนี้จริงๆมันอยู่ที่ ความลำบากในการเปรียบเทียบปริมาณครับ เลือกเครื่องมือดีๆครับ
ตอบพร้อมอ้างอิงข้อความนี้
  #4  
Old 26 พฤศจิกายน 2016, 22:20
i^i i^i ไม่อยู่ในระบบ
กระบี่ไว
 
วันที่สมัครสมาชิก: 02 มีนาคม 2014
ข้อความ: 230
i^i is on a distinguished road
Default

อ้างอิง:
ข้อความเดิมเขียนโดยคุณ Aquila View Post
โจทย์มันอยากได้ความสัมพันธ์ของ "ค่าเลขชี้กำลังสูงสุด" (ที่เจอใน $m!$)

กับ "จำนวน $m$" จริงป่าวครับ เพราะฉะนั้นพยายามหาเครื่องมือมา "เปรียบเทียบ"

ค่าของเลขชี้กำลังสูงสุดกับค่าของ value $m$

พิสูจน์ให้ได้ก่อนว่าค่ามากที่สุดของเลขชี้กำลัง มันจะเจอใน $p_{1}=2$ ครับ

แล้วใช้เครื่องมือบางอย่าง "โยง" ข้อมูลของ $n_{1}$ (ในตำแหน่ง $p=2$)

เข้าไปเปรียบเทียบกับค่าของ $m$ ครับ

หรือง่ายๆคือ สร้างความสัมพันธ์ของเลขชี้กำลัง $n_{1}-1$ กับ $m$ ให้เปรียบเทียบค่ากันให้ได้

ผมทดจนสุดๆดูแล้วมันไม่ได้ใช้อุปนัยเชิงคณิตศาสตร์ได้ตรงๆครับ ต้องทอนโจทย์ลงมาก่อน

ถ้าไม่ได้จริงๆ แล้ววิธีผมไม่มีปัญหาเดี๋ยวมาพิมพ์ไว้ให้ทีหลังครับ สู้ๆนะครับ

ปล. ปัญหาข้อนี้จริงๆมันอยู่ที่ ความลำบากในการเปรียบเทียบปริมาณครับ เลือกเครื่องมือดีๆครับ

ขอบคุณมากครับผม. อยากเห็นวิธีคิดสักหน่อยครับ. ^^
ตอบพร้อมอ้างอิงข้อความนี้
  #5  
Old 26 พฤศจิกายน 2016, 22:48
Aquila Aquila ไม่อยู่ในระบบ
บัณฑิตฟ้า
 
วันที่สมัครสมาชิก: 29 ตุลาคม 2013
ข้อความ: 412
Aquila is on a distinguished road
Default

อ้างอิง:
ข้อความเดิมเขียนโดยคุณ i^i View Post
ขอบคุณมากครับผม. อยากเห็นวิธีคิดสักหน่อยครับ. ^^
ทำไปถึงตรงไหนแล้วครับ ? อยากให้ลอง crack ให้ได้เองก่อนอะครับ

ผมว่ามันเป็นโจทย์ที่ดีมากข้อนึงนะครับ หรือผมหลงไปเองว่าวิธีผมมันถูก
ตอบพร้อมอ้างอิงข้อความนี้
  #6  
Old 27 พฤศจิกายน 2016, 01:57
Aquila Aquila ไม่อยู่ในระบบ
บัณฑิตฟ้า
 
วันที่สมัครสมาชิก: 29 ตุลาคม 2013
ข้อความ: 412
Aquila is on a distinguished road
Default

ผมทวน proof ของตัวเองดูแล้วระบบ induction ของผมยังไม่ cover โดเมนครับ

ไอเดียที่ผมทดไว้คือสร้างเครื่องมือโยงค่า max บนเลขชี้กำลัง

ในที่นี้คือ $n_{1}$ ในตำแหน่งที่ $p=2$ (ใช้เลอจองด์พิสูจน์ออกมาว่า max ที่ 2)

จากนั้นก็ใช้ข้อเท็จจริงที่ว่า $n_{1}=\frac{m-\sum a_{i}}{p_{1}-1}$ $0 \leq i \leq k$ (เลอจองด์อีกฟอร์ม)

แต่ prove ไปแล้วว่า max $p$ มันเกิดที่ $p_{1}=2$

เลยได้ว่าต้อง prove $2^{m-\sum a_{i}-1} > m$ โดยที่ $m=a_{k}2^{k}+a_{k-1}2^{k-1}+...+a_{1}2+a_{0}$

เป็น base 2 representation ของ $m$

เพราะงั้น $a_{i}$ เป็นได้แค่ $0,1$ จากนั้นใช้ความรู้อสมการมาหยิบขอบของค่า $\sum a_{i}$ ที่เป็นไปได้

เอาเข้าไปเปรียบเทียบกับค่าของ $m-1$ บนเลขชี้กำลังครับ

แต่เวลาเปรียบเทียบค่ามันเอาไปเทียบ $m$ ตรงๆลำบาก ผมเลยเลือกวิธีตัดเซทของ $m$ ออกเป็นช่วงๆ

ด้วยค่าของจำนวนเป็นคู่ๆ (ที่เลือกมาให้เปรียบเทียบค่าของ $\sum a_{i}$ ได้ )

ก็เท่ากับว่าใช้จำนวนนั้นเป็นเส้นแบ่งโดนเมนของ $\mathbb{N}$ แล้วพิสูจน์ข้อความ for all ใช้ช่วงที่แบ่ง

ตรงนั้นแหละที่ใช้ induction เล็กๆครับ จากนั้นใช้ผลของ subset ของ $\mathbb{N}$ ที่แบ่งไว้

มา union กันให้ cover $\mathbb{N}$ ตามที่โจทย์บอกคือ $m \geq 6$ ก็จบครับ

แต่ขั้นสุดท้าย ผมมาเจอปัญหาซะก่อน ถ้าหากซีเรียสมาก ไว้มีเวลาเดี๋ยวผมมาจัดการให้ครับ

ปล. ขั้นสุดท้ายคือ prove ให้คลุมโดเมนที่โจทย์อยากได้ ผมมีไอเดียเรื่อง Cauchy induction, Strong induction

กับเหตุผลเรื่อง bijection ที่ส่งโดเมนที่แบ่งออกให้สมมูลกับระบบของ induction แต่ยังไม่หลุดครับ
ตอบพร้อมอ้างอิงข้อความนี้
  #7  
Old 27 พฤศจิกายน 2016, 03:08
i^i i^i ไม่อยู่ในระบบ
กระบี่ไว
 
วันที่สมัครสมาชิก: 02 มีนาคม 2014
ข้อความ: 230
i^i is on a distinguished road
Default

อ้างอิง:
ข้อความเดิมเขียนโดยคุณ Aquila View Post
ผมทวน proof ของตัวเองดูแล้วระบบ induction ของผมยังไม่ cover โดเมนครับ

ไอเดียที่ผมทดไว้คือสร้างเครื่องมือโยงค่า max บนเลขชี้กำลัง

ในที่นี้คือ $n_{1}$ ในตำแหน่งที่ $p=2$ (ใช้เลอจองด์พิสูจน์ออกมาว่า max ที่ 2)

จากนั้นก็ใช้ข้อเท็จจริงที่ว่า $n_{1}=\frac{m-\sum a_{i}}{p_{1}-1}$ $0 \leq i \leq k$ (เลอจองด์อีกฟอร์ม)

แต่ prove ไปแล้วว่า max $p$ มันเกิดที่ $p_{1}=2$

เลยได้ว่าต้อง prove $2^{m-\sum a_{i}-1} > m$ โดยที่ $m=a_{k}2^{k}+a_{k-1}2^{k-1}+...+a_{1}2+a_{0}$

เป็น base 2 representation ของ $m$

เพราะงั้น $a_{i}$ เป็นได้แค่ $0,1$ จากนั้นใช้ความรู้อสมการมาหยิบขอบของค่า $\sum a_{i}$ ที่เป็นไปได้

เอาเข้าไปเปรียบเทียบกับค่าของ $m-1$ บนเลขชี้กำลังครับ

แต่เวลาเปรียบเทียบค่ามันเอาไปเทียบ $m$ ตรงๆลำบาก ผมเลยเลือกวิธีตัดเซทของ $m$ ออกเป็นช่วงๆ

ด้วยค่าของจำนวนเป็นคู่ๆ (ที่เลือกมาให้เปรียบเทียบค่าของ $\sum a_{i}$ ได้ )

ก็เท่ากับว่าใช้จำนวนนั้นเป็นเส้นแบ่งโดนเมนของ $\mathbb{N}$ แล้วพิสูจน์ข้อความ for all ใช้ช่วงที่แบ่ง

ตรงนั้นแหละที่ใช้ induction เล็กๆครับ จากนั้นใช้ผลของ subset ของ $\mathbb{N}$ ที่แบ่งไว้

มา union กันให้ cover $\mathbb{N}$ ตามที่โจทย์บอกคือ $m \geq 6$ ก็จบครับ

แต่ขั้นสุดท้าย ผมมาเจอปัญหาซะก่อน ถ้าหากซีเรียสมาก ไว้มีเวลาเดี๋ยวผมมาจัดการให้ครับ

ปล. ขั้นสุดท้ายคือ prove ให้คลุมโดเมนที่โจทย์อยากได้ ผมมีไอเดียเรื่อง Cauchy induction, Strong induction

กับเหตุผลเรื่อง bijection ที่ส่งโดเมนที่แบ่งออกให้สมมูลกับระบบของ induction แต่ยังไม่หลุดครับ
ขอบคุณมากครับ. เเต่ผมพึ่งจะเริ่มศึกษาอุปนัยฯอะครับ. ยังไม่ได้ลงลึกเท่านี้เลยครับ.
ต้องขอคำเเนะนำด้วยครับ. จะพยายามทำความเข้าใจครับ.
ตอบพร้อมอ้างอิงข้อความนี้
  #8  
Old 27 พฤศจิกายน 2016, 03:21
Aquila Aquila ไม่อยู่ในระบบ
บัณฑิตฟ้า
 
วันที่สมัครสมาชิก: 29 ตุลาคม 2013
ข้อความ: 412
Aquila is on a distinguished road
Default

อ้างอิง:
ข้อความเดิมเขียนโดยคุณ i^i View Post
ขอบคุณมากครับ. เเต่ผมพึ่งจะเริ่มศึกษาอุปนัยฯอะครับ. ยังไม่ได้ลงลึกเท่านี้เลยครับ.
ต้องขอคำเเนะนำด้วยครับ. จะพยายามทำความเข้าใจครับ.
ถ้าเป็นแบบนี้ผมว่า ผมคิดอะไรเลอะเทอะเกินไปแล้วละ

อาจจะมีวิธีดีๆกว่านี้ก็ได้ครับ อีกอย่างวิธีของผมมันเป็น induction ย่อยๆอีกที

ลองเปิดแหล่งโจทย์ดูครับ โจทย์ข้อนี้มาจากไหน อาจจะมีวิธีทีมองๆข้ามไปที่ง่ายกว่าก็ได้ครับ

ปล. induction ไม่ได้มีแค่แบบเดียวครับ induction แบบแบ่งช่วงก็มีครับ

ผมขอเบรคไว้ตรงนี้ก่อน ตอนนี้ติดภาระเกี่ยวพันครับ

ลองดูข้างล่างครับ insight เรื่อง induction ดีๆ เพื่อได้ไอเดียอะไรใหม่ๆ

http://math.stackexchange.com/questi...edirect=1&lq=1
ตอบพร้อมอ้างอิงข้อความนี้
  #9  
Old 27 พฤศจิกายน 2016, 12:49
i^i i^i ไม่อยู่ในระบบ
กระบี่ไว
 
วันที่สมัครสมาชิก: 02 มีนาคม 2014
ข้อความ: 230
i^i is on a distinguished road
Default

อ้างอิง:
ข้อความเดิมเขียนโดยคุณ Aquila View Post
ถ้าเป็นแบบนี้ผมว่า ผมคิดอะไรเลอะเทอะเกินไปแล้วละ

อาจจะมีวิธีดีๆกว่านี้ก็ได้ครับ อีกอย่างวิธีของผมมันเป็น induction ย่อยๆอีกที

ลองเปิดแหล่งโจทย์ดูครับ โจทย์ข้อนี้มาจากไหน อาจจะมีวิธีทีมองๆข้ามไปที่ง่ายกว่าก็ได้ครับ

ปล. induction ไม่ได้มีแค่แบบเดียวครับ induction แบบแบ่งช่วงก็มีครับ

ผมขอเบรคไว้ตรงนี้ก่อน ตอนนี้ติดภาระเกี่ยวพันครับ

ลองดูข้างล่างครับ insight เรื่อง induction ดีๆ เพื่อได้ไอเดียอะไรใหม่ๆ

http://math.stackexchange.com/questi...edirect=1&lq=1

ขอบคุณมากครับ. ผมจะพยายามศึกษาครับ ^^
ตอบพร้อมอ้างอิงข้อความนี้
  #10  
Old 28 พฤศจิกายน 2016, 16:53
Thamma Thamma ไม่อยู่ในระบบ
ลมปราณคุ้มครองร่าง
 
วันที่สมัครสมาชิก: 19 กุมภาพันธ์ 2013
ข้อความ: 307
Thamma is on a distinguished road
Default

ตอบพร้อมอ้างอิงข้อความนี้
  #11  
Old 28 พฤศจิกายน 2016, 17:58
Aquila Aquila ไม่อยู่ในระบบ
บัณฑิตฟ้า
 
วันที่สมัครสมาชิก: 29 ตุลาคม 2013
ข้อความ: 412
Aquila is on a distinguished road
Default

อ้างอิง:
ข้อความเดิมเขียนโดยคุณ Thamma View Post
บทพิสูจน์ตัวนี้สวยและคลีนมากจริงๆครับ เยี่ยมเลย
ตอบพร้อมอ้างอิงข้อความนี้
  #12  
Old 29 พฤศจิกายน 2016, 05:11
i^i i^i ไม่อยู่ในระบบ
กระบี่ไว
 
วันที่สมัครสมาชิก: 02 มีนาคม 2014
ข้อความ: 230
i^i is on a distinguished road
Default

อ้างอิง:
ข้อความเดิมเขียนโดยคุณ Thamma View Post

ขอบคุณครับผม ^^
ตอบพร้อมอ้างอิงข้อความนี้
  #13  
Old 10 ธันวาคม 2016, 15:00
Aquila Aquila ไม่อยู่ในระบบ
บัณฑิตฟ้า
 
วันที่สมัครสมาชิก: 29 ตุลาคม 2013
ข้อความ: 412
Aquila is on a distinguished road
Default

ไปคิดมาให้แล้วแบบจริงๆจังๆครับ ข้อนี้ไม่ยากเท่าไร อสมการ weak เลยทำได้หลายวิธี

ทดไว้ประมาณ 5-6 วิธีแต่เอาแค่ 4 ก่อน ผิดตรงไหนหรือมีวิธีแปลกๆก็รบกวนด้วยนะครับ

เริ่มที่ Lemma ที่ผมใช้ไปก่อน

Legendre's Lemma
ให้ $p \in \mathbb{P}$ และ $m \in \mathbb{N}$ จะได้ว่าจำนวนเต็มบวก $e$ ที่ใหญ่สุดที่ทำให้
$p^e$ fully divide $m!$ คือ $e=v_{p}(m!)=\sum_{i = 1}^{\infty} \left\lfloor\,\frac{m}{p^i}\right\rfloor$

Lemma 1
จาก Lemma บนจะได้ว่า ถ้า $\sum_{i = 1}^{\infty} \left\lfloor\,\frac{m}{p^i}\right\rfloor =\sum_{i = 1}^{j} \left\lfloor\,\frac{m}{p^i}\right\rfloor$ แล้ว $j=\left\lfloor\,\log_p{m}\right\rfloor$
สมมติให้ $j$ เป็นจำนวนเต็มบวกใหญ่สุดที่ทำให้ $\left\lfloor\,\frac{m}{p^j}\right\rfloor$ ไม่เป็น $0$ และ $\left\lfloor\,\frac{m}{p^{j+1}}\right\rfloor=0$ ทุกค่าเลขชี้กำลังที่ใหญ่กว่า
และจากการที่ $ \left\lfloor\,x\right\rfloor \leq x \leq \left\lfloor\,x\right\rfloor +1 $
ใช้อสมการมาวิเคราะห์ข้างในจะพิสูจน์ได้ว่า $\left\lfloor\,\log_p{m}\right\rfloor -1 \leq \log_p{m} -1 < j < \log_p{m} < \left\lfloor\,\log_p{m}\right\rfloor +1$
จากอสมการคู่ซ้ายและขวาสุด ต้องบีบได้เป็น $j=\left\lfloor\,\log_p{m}\right\rfloor$

Lemma 2
$v_{p}(m!)=u+ v_{p}(u!)$ โดยที่ $u= \left\lfloor\,\frac{m}{p}\right\rfloor$
สมมติให้ $u$ เป็นจำนวนเต็มบวกใหญ่ที่สุดที่ทำให้ $up \leq m$ แต่ $(u+1)p > m$
ให้ $m!=p^u u! u'$ โดยที่ $(p,u')=1$ จะได้ว่า $v_{p}(m!)=u+v_{p}(u!)$
แต่จากอสมการชุดบนทำให้ $u \leq \frac{m}{p} < u+1$ จะได้ว่า $\left\lfloor\,\frac{m}{p}\right\rfloor = u$
------------------------------------------------------------------------
note

ต่อจากนี้ ให้ระวังแต่ละวิธีด้วย ว่าบทพิสูจน์นั้นๆ cover ตั้งแต่ $m$ เป็นเท่าไร

อันนี้เขียนให้เหตุผลแยกเอาได้ครับ ผมขี้เกียจเพราะมันเยอะ ที่ทดไว้มันไม่ได้เริ่มที่ $m \geq 6$

แต่มันสามารถแยกเชคได้ง่ายๆครับ เริ่มเลย

ต้องพิสูจน์ก่อนว่าค่า $n_{i}$ ใหญ่สุดที่ดึงเอาจาก $m!$ คือเกิดขึ้นที่ $p=2$ ตามความเห็นบน

ต่อจากนี้ก็แล้วแต่คน จะทำได้หลายๆวิธี
-----------------------------------------------------------------------
วิธีที่ 1

พิสูจน์โดยอุปนัยเชิงคณิตศาสตร์ได้ไม่ยากว่า ถ้า $n \geq 3$ แล้ว $2^n \geq 2n+2$

เป็นการเพียงพอที่จะพิสูจน์ว่า $2^{n_{1}-1} \geq 2(n_{1}-1)+2 \geq m+1$

หรือ $2n_{1} \geq m+1$ จาก Lemma และที่พิสูจน์ไปแล้ว จะได้ว่า $n_{1}=\left\lfloor\,\frac{m}{2}\right\rfloor + v_{2}(\left\lfloor\,\frac{m}{2}\right\rfloor !)$

ดังนั้นต้องพิสูจน์ว่า $2n_{1}=2\left\lfloor\,\frac{m}{2}\right\rfloor +2v_{2}(\left\lfloor\,\frac{m}{2}\right\rfloor!) \geq m+1$

จากอสมการ $\left\lfloor\,x\right\rfloor \leq x < \left\lfloor\,x \right\rfloor +1 $ และจากการที่ $ v_{2}(\left\lfloor\,\frac{m}{2}\right\rfloor!) \geq v_{2}(\left\lfloor\,\frac{6}{2}\right\rfloor!) = v_{2}(3!)=1$

จะได้ว่า $2\left\lfloor\,\frac{m}{2}\right\rfloor +2v_{2}(\left\lfloor\,\frac{m}{2}\right\rfloor!) > 2(\frac{m}{2}-1)+2\cdot 1 \geq m+1 > m$ ตามต้องการ
----------------------------------------------------------------------------------
วิธีที่ 2

จะพิสูจน์อสมการที่แข็งกว่าวิธีที่ 1 ดังนี้ $2^n \geq n^2 \geq 2n+2 \geq m+1$

จริงเมื่อ $n \geq ... m \geq ...$ (อันนี้ define ได้) ...เวลาจะเขียนอีก 1 solution...

พิสูจน์โดยไม่ยากว่า $2^n \geq n^2$ ดังนั้นเป็นการเพียงพอจะ prove $2^{n_{1}-1} \geq (n_{1}-1)^2 \geq 2n_{1} \geq m+1$

(เลือกเอา $(n_{1}-1)^2 \geq m+1$ ไปใช้เลย)

กระจายแล้วจัดรูปจะได้ว่า $n_{1} \geq 1+\log \sqrt{m+1}$ หรือ $10^{\left\lfloor\,\frac{m}{2}\right\rfloor +2v_{2}(\left\lfloor\,\frac{m}{2}\right\rfloor!)-1} \geq \log \sqrt{m+1}$

แต่จาก AM-GM จะได้ $\log \sqrt{m+1} < 1+\frac{m}{2}$ และจาก Bernoulli's จะได้ $(1+9)^{\left\lfloor\,\frac{m}{2}\right\rfloor +v_{2}(\left\lfloor\,\frac{m}{2}\right\rfloor!)-1} \geq 1+9(\left\lfloor\,\frac{m}{2}\right\rfloor +v_{2}(\left\lfloor\,\frac{m}{2}\right\rfloor!)-1)$

ดังนั้นเหลือแค่โชว์ $9\left\lfloor\,\frac{m}{2}\right\rfloor +9v_{2}(\left\lfloor\,\frac{m}{2}\right\rfloor!) > 9 +\frac{m}{2} = 9+\left\lfloor\,\frac{m}{2}\right\rfloor +f(\frac{m}{2})$

โดยที่ $f(x)$ คือเลขทศนิยมของ $x$ อสมการบนสุดจริงชัดเจนเพราะ $f(\frac{m}{2}) < 1$

-----------------------------------------------------------------------------------

วิธีที่ 3

จากวิธีอื่นๆ $2^{n_{1}-1} \geq 2n_{1} \geq m+1$

จะ bound ค่าของ $m+1$ กับ $2n_{1}$ มาชนกัน โดยใช้ $\left\lfloor\,x \right\rfloor +\left\lfloor\,y \right\rfloor \leq \left\lfloor\, x+y \right\rfloor \leq \left\lfloor\, x+y \right\rfloor +1 $

$m+1=\frac{m-1}{2}+\frac{m+3}{2} < \left\lfloor\, \frac{m-1}{2}+\frac{m+3}{2} \right\rfloor +1 \leq \left\lfloor\,\frac{m-1}{2}\right\rfloor +\left\lfloor\,\frac{m+3}{2}\right\rfloor +2 = \left\lfloor\,\frac{m-1}{2}\right\rfloor +\left\lfloor\,\frac{m+1}{2}\right\rfloor +3$

อีกข้างก็ทำคล้ายๆกัน

$2n_{1}=2\left\lfloor\,\frac{m}{2} \right\rfloor + v_{2}(\left\lfloor\,\frac{m}{2}\right\rfloor!) \geq 2\left\lfloor\,\frac{m+1}{2}\right\rfloor + 2\left\lfloor\, \frac{m-1}{2}\right\rfloor + v_{2}(\left\lfloor\,\frac{m}{2}\right\rfloor!) \geq \left\lfloor\,\frac{m+1}{2}\right\rfloor + \left\lfloor\, \frac{m-1}{2}\right\rfloor +3$

ดูอสมการสุดท้าย $\left\lfloor\,\frac{m+1}{2}\right\rfloor + \left\lfloor\, \frac{m-1}{2}\right\rfloor + v_{2}(\left\lfloor\,\frac{m}{2}\right\rfloor!) \geq 3$ ซึ่งก็ได้ว่าจริง

----------------------------------------------------------------------------------

วิธีที่ 4

ให้ $y=2^x$ พิสูจน์ได้ไม่ยากว่า $f(\frac{x+y}{2}) \geq \frac{f(x)+f(y)}{2}$

เลือกให้ $x=2\left\lfloor\, \frac{m}{2} \right\rfloor -1$ และ $y=2v_{2}(\left\lfloor\,\frac{m}{2}\right\rfloor!) -1$

จะได้ $2^{n_{1}-1}=2^{\frac{x+y}{2}} \geq 2^{2\left\lfloor\, \frac{m}{2} \right\rfloor -2}+2^{2v_{2}(\left\lfloor\,\frac{m}{2}\right\rfloor!)-2}$

ใช้ Bernoulli's เทอมขวาล่าสุดจะได้ว่าต้องพิสูจน์ $2\left\lfloor\,\frac{m}{2}\right\rfloor +2v_{2}(\left\lfloor\,\frac{m}{2}\right\rfloor!) \geq m+3$

ซึ่งจริงจาก $\left\lfloor\,\frac{m}{2}\right\rfloor \geq \frac{m}{2}-\frac{1}{2}$ และ $v_{2}(\left\lfloor\,\frac{m}{2}\right\rfloor!) \geq 2$

อสมการแรกเขียนใหม่ได้เป็น $f(\frac{m}{2}) \leq \frac{1}{2}$ ซึ่งก็จริงนั่นแหละ

ปล.วิธีที่ 5 มันซับซ้อนขึ้นไปอีกหน่อย ใช้แบ่ง set แบบ $2^k+r$ กับ Lemma 1

เดี๋ยวมาโชว์ให้ดูทีหลังครับ เหนื่อย
ตอบพร้อมอ้างอิงข้อความนี้
  #14  
Old 10 ธันวาคม 2016, 18:24
kongp kongp ไม่อยู่ในระบบ
ลมปราณไร้สภาพ
 
วันที่สมัครสมาชิก: 05 พฤษภาคม 2006
ข้อความ: 1,127
kongp is on a distinguished road
Default

สมาคมคณิตศาสตร์อเมริกา เฉลยสูครจำนวนเฉพาะ แล้วครับ ไม่ต้องรันคอมพิวเตอร์หาจำนวนเฉพาะ ก็ได้ !!!
ตอบพร้อมอ้างอิงข้อความนี้
  #15  
Old 10 ธันวาคม 2016, 18:28
kongp kongp ไม่อยู่ในระบบ
ลมปราณไร้สภาพ
 
วันที่สมัครสมาชิก: 05 พฤษภาคม 2006
ข้อความ: 1,127
kongp is on a distinguished road
Default

น่ากลัวอยู่ ถ้าวิชา และ ตำราที่เกี่ยวกับ Number Theory ในโลกนี้จะหายไป ...

คงเหลือแต่ Data Set Disk ขายกัน เหมือนๆ แบบบ้าน ร้านค้า แบบโรงงาน
ตอบพร้อมอ้างอิงข้อความนี้
ตั้งหัวข้อใหม่ Reply



กฎการส่งข้อความ
คุณ ไม่สามารถ ตั้งหัวข้อใหม่ได้
คุณ ไม่สามารถ ตอบหัวข้อได้
คุณ ไม่สามารถ แนบไฟล์และเอกสารได้
คุณ ไม่สามารถ แก้ไขข้อความของคุณเองได้

vB code is On
Smilies are On
[IMG] code is On
HTML code is Off
ทางลัดสู่ห้อง


เวลาที่แสดงทั้งหมด เป็นเวลาที่ประเทศไทย (GMT +7) ขณะนี้เป็นเวลา 19:21


Powered by vBulletin® Copyright ©2000 - 2024, Jelsoft Enterprises Ltd.
Modified by Jetsada Karnpracha